Entwicklung von Eigenzuständen bei Kopplung zweier Spinsysteme

Ich möchte folgende Situation schildern:

Wir haben zwei Spinsysteme: Spin 1 ( S 1 ) und Schleudern 1/2 ( S 2 ).

Stellen Sie sich nun vor, Sie ändern irgendwie ihre Interaktion, damit Sie die Kopplung feinabstimmen können J zwischen ihnen in der Form:

H = S 1 J 12 S 2

Wo J ist eine Matrix, die diese Interaktion beschreibt.

Jetzt ist meine Frage, wie ich das in Matrixform schreibe, um die verschiedenen Eigenzustände dieses gekoppelten Systems für verschiedene Kopplungsstärken zu berechnen J ?

Soll ich ein Spin-3/2-System (4x4-Matrix) oder einen verschränkten Hilbert-Raum mit Spin 1/2 und Spin 1 (6x6-Matrix) annehmen?

Was ist auch, wenn ich noch Effekte auf das Spin-1-System wie die Zeeman-Aufspaltung in einem Magnetfeld einbeziehen möchte B z , wie könnte ich das einbinden?


Aktualisieren

Machen wir die Situation also etwas einfacher, nur ein Magnetfeld B z wirkt auf den Spin-1 und nur eine isotrope ferromagnetische Kopplung zwischen Spin-1 und Spin-1/2:

H = G μ B B z S z + J S S P ich N 1 S S P ich N 1 / 2

Ich kenne also meine Spinmatrizen für den Spin-1/2 (Pauli-Matrizen) und für den Spin-1. Mein Ansatz wäre jetzt, das Tensorprodukt dieser Operatoren zu nehmen, um die neuen Operatoren für den obigen Hamilton-Operator zu erstellen, dh:

S X B Ö T H = S X S P ich N 1 S X S P ich N 1 / 2
sowie für j Und z .

Damit konstruiere ich den neuen Hamilton-Operator, ich denke, diese Operatoren sind für den Spin-Kopplungsterm richtig, für das Magnetfeld B_z, das nur auf den Spin-1 wirken soll, muss ich es auf den Unterraum des Spin-1-Systems projizieren, denke ich ?

Liebe Matthias. Ich habe vor, auf meine Antwort zurückzukommen, um sie ein wenig aufzuräumen und sie irgendwann allgemeiner zu machen. Ich habe am Ende einige Notizen gemacht, wie Sie das Problem angehen könnten: Ich vermute, der beste Weg ist, Schurs Lemma auf irgendeine Weise zu verwenden. Sie könnten auch den Unterraum finden, der allen drei Nullräumen der drei gemeinsam ist 36 × 36 Matrizen 1 36 × 36 Σ J Σ J T 1 36 × 36 in Mathematica oder Matlab, aber ich vermute, es gibt eine viel elegantere Methode.
Bitte fügen Sie auch Ihre eigene Antwort hinzu, wenn Sie es herausfinden: Ich interessiere mich jetzt selbst sehr dafür. Meine Antwort könnte die Frage wahrscheinlich so umformulieren, dass sie in Maths SE gestellt werden kann.
Lieber Rod, danke für die ausführliche Antwort, aber ich fürchte, das ist mir etwas zu kompliziert. Ich dachte, es muss einfacher sein, indem ich ein paar Vereinfachungen mache, zB kümmern wir uns nur um den isotropen Teil der Kopplung und gehen von einer ferromagnetischen Kopplung aus. Nehmen wir also an, in diesem Fall möchte ich ein externes Magnetfeld B_z an das Spin-1-System anlegen und die beiden Spins werden durch verbunden H = J S 1 S 2 . Unser gesamter Hamiltonoperator wäre also: H = G μ B z S z + J S 1 S 2 . Kann ich nicht einfach die Tensorprodukte der einzelnen Operatoren nehmen und das war's?

Antworten (1)

Ich habe noch nie darüber nachgedacht, also ist hier ein Ansatz, der funktionieren wird, wenn Sie hart genug daran arbeiten.

Bevor ich anfange zu hämmern, Punkt Nummer 1:

Soll ich ein Spin-3/2-System (4x4-Matrix) oder einen verschränkten Hilbert-Raum mit Spin 1/2 und Spin 1 (6x6-Matrix) annehmen?

Zweifellos letzteres. Es ist ein zweiteiliges System und sein Zustandsraum ist das Tensorprodukt der beiden Teilchenräume. Es kann einfach nichts anderes sein.

Das Grundprinzip hier ist die Erhaltung des Drehimpulses, also ist Ihr grundlegendes Verfahren zur Lösung Ihres Problems:

  1. Erarbeiten Sie die Matrizen für die Observablen der drei Nettodrehimpulskomponenten (die drei Nettodrehimpulsoperatoren);

  2. Finden Sie den allgemeinsten Hamilton-Operator, der für alle drei kommutiert, da die Kommutierung mit dem Hamilton-Operator der zeitlichen Invarianz aller Momente der Wahrscheinlichkeitsverteilungen der Messungen entspricht.

Teil 1: Die drei Drehimpulsoperatoren

Der X -AM-Komponente beobachtbar für das Spinhalbteilchen,

σ X = ( 0 1 1 0 )

hat AM-Eigenvektoren:

ψ + = 1 2 ( 1 1 ) ; ψ = 1 2 ( 1 1 )

und AM-Eigenwerte λ + = + 1 2 Und λ = 1 2 , bzw.

Der X -AM-Komponente beobachtbar für das Spin-1-Teilchen,

S X = ( 0 0 0 0 0 ich 0 ich 0 )

hat AM-Eigenvektoren:

Ψ + = 1 2 ( 0 ich 1 ) ; Ψ = 1 2 ( 0 1 ich ) ; Ψ 0 = ( 1 0 0 )

und AM-Eigenwerte Λ + = + 1 , Λ = 1 Und Λ 0 = 0 , bzw. Also jetzt, für das Zwei-Teilchen-System, die Sechs X -AM-Eigenzustände sind:

  1. ψ + Ψ + mit AM-Eigenwert 1 2 + 1 = 3 2
  2. ψ + Ψ 0 mit AM-Eigenwert 1 2 + 0 = 1 2
  3. ψ + Ψ 1 mit AM-Eigenwert 1 2 1 = 1 2
  4. ψ Ψ + mit AM-Eigenwert 1 2 + 1 = 1 2
  5. ψ Ψ 0 mit AM-Eigenwert 1 2 + 0 = 1 2
  6. ψ Ψ 1 mit AM-Eigenwert 1 2 1 = 3 2

Wenn wir also die Eigenzustände wie oben anordnen, sind die Eigenvektoren als Spalten v e C ( ψ + Ψ + ) , v e C ( ψ + Ψ 0 ) (siehe die Wikipedia-Vektorisierungsseite ) und so erhalten wir schließlich die Summe X -AM-Komponente beobachtbar Σ X = P X Λ X P X Wo

P X = ( 0 1 2 0 0 1 2 0 0 1 2 0 0 1 2 0 ich 2 0 1 2 ich 2 0 1 2 ich 2 0 1 2 ich 2 0 1 2 1 2 0 ich 2 1 2 0 ich 2 1 2 0 ich 2 1 2 0 ich 2 )

Und Λ X = D ich A G ( 3 2 , 1 2 , 1 2 , 1 2 , 1 2 , 3 2 ) . Das Ergebnis ist:

Σ X = ( 0 1 2 0 0 0 0 1 2 0 0 0 0 0 0 0 3 4 1 4 ich 4 3 ich 4 0 0 1 4 3 4 3 ich 4 ich 4 0 0 ich 4 3 ich 4 3 4 1 4 0 0 3 ich 4 ich 4 1 4 3 4 )

Ab hier sollte konzeptionell klar sein, wie es weitergeht, wenn auch mühsam. Dasselbe machst du für die j -AM-Beobachtbare:

σ j = ( 0 ich ich 0 )
S j = ( 0 0 ich 0 0 0 ich 0 0 )

um das Gesamtsystem zu finden j -AM beobachtbar Σ Y und für die z -AM-Beobachtbare:

σ z = ( ich 0 0 ich )
S z = ( 0 ich 0 ich 0 0 0 0 0 )

um das Gesamtsystem zu erhalten z -AM beobachtbar Σ Z .

Teil 2: Finden Sie den allgemeinsten Hamiltonoperator

Ihr allgemeinster Hamiltonoperator wird durch die drei Kommutatorbeziehungen definiert, die die Erhaltung von AM ausdrücken:

[ H ^ , Σ J ] = 0 ; J = X , Y , Z

Sie müssen die unveränderlichen Räume der drei ausarbeiten Σ s dazu. Sie erhalten einen linearen Raum möglich H ^ s: Im Fall zweier gekoppelter Spinhalbteilchen gibt es im Wesentlichen nur einen möglichen Hamiltonoperator, der aus diesem Ansatz herausfällt, und der proportional zu ist σ X σ X + σ j σ j + σ z σ z (plus einen Term proportional zum 4 × 4 Identitätsmatrix, die die Verschiebung der Grundzustandsenergie ausdrückt), aber in diesem sechsdimensionalen Fall werden die Dinge etwas komplizierter. Nun, wie gesagt, ich habe das noch nie zuvor gemacht, also wage ich zu behaupten, dass es einen systematischeren und weniger umständlichen Weg gibt, das alles zu lösen. Aber jede Methode wird auf den oben genannten ersten Prinzipien beruhen.

Magnetfeld

Wie lauten die Begriffe für den Einfluss des Magnetfelds? Nun, das ist einfach: In der Reihenfolge, die wir oben studiert haben, wird der ungekoppelte Hamilton-Operator sein:

H ^ = γ 1 2 ( σ X B X + σ j B j + σ z B z ) 1 3 × 3 + γ 1 ICH 2 × 2 ( S X B X + S j B j + S z B z )

Wo γ 1 2 Und γ 1 sind die jeweiligen gyromagnetischen Verhältnisse.


Hinweise zur Durchführung der Methode. Sie können auch einen Zweiparteienstaat darstellen Φ = ψ Ψ als wörtlich 2 × 3 Matrix, die das äußere Produkt ist Φ = ψ P S ich T des 2 × 1 Und 3 × 1 Spaltenvektoren. Dann agieren die Operatoren auf dem ersten Feld links und die Operatoren auf dem zweiten Feld rechts. So unser X -Komponente Observable wäre die lineare, homogene Transformation:

Φ σ X Φ S X T

und der Vektorisierungsoperator (Siehe Vectorization Wiki Page) , der unsere Zustände in a umordnet 6 × 1 Spaltenvektoren wie in meiner Antwort, schreibt dies als

v e C ( Φ ) S X σ X v e C ( Φ )

Mit der Standardformel v e C ( A B C ) = C T A v e C ( B ) . Anhand der Formel ( A B ) ( C D ) = ( A C ) ( B D ) , und unter Verwendung der Tatsache, dass inverse, komplex konjugierte, hermitesch konjugierte und transponierte Operationen sich über das Kronecker-Produkt verteilen, können wir diagonalisieren S X σ X innerhalb des Kronecker-Produkts und stellen Sie fest, dass die Eigenzustände des gekoppelten Systems sind Π X π X , Wo P X , P X sind die als Spalten geschriebenen Matrizen der Eigenvektoren der einzelnen Multiplikanden. Damit können Sie rechnen Σ J , J = X , Y , Z systematisch und schnell.

Um nun den allgemeinsten Hamilton-Operator zu finden, müssen Sie den invarianten Raum der Gruppe von Matrizen finden, die von den drei Matrizen erzeugt werden exp ( ich Σ J ) und finden Sie die irreduzible Darstellung davon: äquivalent den kleinsten Vektorunterraum von C 6 von der Gruppe invariant gelassen: Nach Schurs Lemma muss jede Matrix, die mit allen dreien pendelt, proportional zum Identitätsoperator sein, wenn sie auf diesen Unterraum beschränkt ist. Der Skalierungsfaktor ist möglicherweise Null – dh der Operator könnte möglicherweise der Null-Endomorphismus sein. Dies charakterisiert den allgemeinsten Hamiltonian vollständig: Es kann jeder Operator sein, der proportional zur Identität ist, wenn er auf diesen irreduziblen Unterraum beschränkt ist.

Sie könnten auch den Unterraum finden, der allen drei Nullräumen der drei gemeinsam ist 36 × 36 Matrizen 1 36 × 36 Σ J Σ J T 1 36 × 36 in Mathematica oder Matlab, aber ich vermute, dass es eine viel elegantere Methode gibt, die auf Schurs Lemma basiert!